You are on page 1of 4

Volume 15, Number 3 October 2010 - December, 2010

Olympiad Corner IMO Shortlisted Problems


Below are the problems of the 2010
Chinese Girls’ Math Olympiad, which Kin Y. Li
was held on August 10-11, 2010.

Problem 1. Let n be an integer greater Every year, before the IMO begins, Using AiAi+3 is parallel to Ai+1Ai+2, by
than two, and let A1, A2, …, A2n be a problem selection committee collects subtracting coordinates, we can see Bi
pairwise disjoint nonempty subsets of problem proposals from many nations. ≠Bi+1 and BiBi+1 is parallel to Ai+1Ai+2.
{1,2,…,n}. Determine the maximum Then it prepares a short list of problems By symmetry, B1B2⋯Bn is a regular
for the leaders to consider when the n-gon inside A1A2⋯An. Hence, the side
value of ∑ | Ai ∩ Ai +1 | . (Here we set
2n

leaders meet at the IMO site. The length of B1B2⋯Bn is less than the side
i =1 | Ai | ⋅ | Ai +1 |
following were some of the interesting length of A1A2⋯An. This contradicts the
A2n+1=A1. For a set X, let |X| denote the shortlisted problems in past years that side length of A1A2⋯An is supposed to
number of elements in X.) were not chosen. Perhaps some of the be minimal. Therefore, n=4 is the only
Problem 2. In ⊿ABC, AB=AC. Point ideas may reappear in later proposals in possible case.
D is the midpoint of side BC. Point E coming years.
lies outside ⊿ABC such that CE⊥AB Example 2. (1987 IMO Proposal by
Example 1. (1985 IMO Proposal by
and BE=BD. Let M be the midpoint of Yugoslavia) Prove that for every natural
Israel) For which integer n ≥ 3 does
segment BE. Point F lies on the minor number k (k ≥ 2) there exists an
there exist a regular n-gon in the plane
arc AD of the circumcircle of ⊿ABD irrational number r such that for every
such that all its vertices have integer
such that MF⊥BE. Prove that ED⊥FD. natural number m,
coordinates in a rectangular coordinate
system? [rm] ≡ −1 (mod k).
A
Solution. Let Ai have coordinates (xi,yi), (Here [x] denotes the greatest integer
F where xi, yi are integers for i=1,2,⋯,n. less than or equal to x.)
In the case n = 3, if A1A2A3 is equilateral,
then on one hand, its area is (Comment: The congruence equation is
E equivalent to [rm]+1 is divisible by k.
Since [rm]≤ rm < [rm] +1, we want to add
M 4
3
A1 A22 =
4
3
( )
( x1 − x 2 ) 2 + ( y1 − y 2 ) 2 ,
a small amount δ ∈(0,1] to rm to make it
B D C which is irrational. On the other hand, an integer divisble by k. If we can get δ
(continued on page 4) its area is also = sm for some s∈(0,1), then some
A1 A2 × A1 A3 algebra may lead to a solution.)
1 x2 − x1 y2 − y1
Editors: 張 百 康 (CHEUNG Pak-Hong), Munsang College, HK
=± ,
2 2 x3 − x1 y3 − y1 Solution. If I have a quadratic equation
高 子 眉 (KO Tsz-Mei)
梁 達 榮 (LEUNG Tat-Wing) which is rational. Hence, the case n = 3 f(x) = x2−akx+bk = 0
李 健 賢 (LI Kin-Yin), Dept. of Math., HKUST
leads to contradiction. The case n = 4 is
吳 鏡 波 (NG Keng-Po Roger), ITC, HKPU with a, b integers and irrational roots r
true by taking (0,0),(0,1),(1,1) and (1,0).
Artist: 楊 秀 英 (YEUNG Sau-Ying Camille), MFA, CU and s such that s∈(0,1), then r+s=ak
The case n = 6 is false since A1A3A5
Acknowledgment: Thanks to Elina Chiu, Math. Dept., ≡0(mod k) and rs=bk≡0(mod k). Using
HKUST for general assistance.
would be equilateral.
rm+1+sm+1=(r+s)(rm+sm)−rs(rm−1+sm−1),
On-line: For the other cases, suppose A1A2⋯An is
http://www.math.ust.hk/mathematical_excalibur/
such a regular n-gon with minimal side by induction on m, we see rm+sm is also
The editors welcome contributions from all teachers and an integer as cases m=0,1 are clear. So
students. With your submission, please include your name,
length. For i=1,2,⋯,n, define point Bi so
address, school, email, telephone and fax numbers (if that AiAi+1Ai+2Bi is a parallelogram
available). Electronic submissions, especially in MS Word, (where An+1=A1 and An+2=A2). Since [rm] +1= rm+sm ≡ (r+s)m ≡ 0 (mod k).
are encouraged. The deadline for receiving material for the
next issue is January 14, 2011. Ai+1Ai+2 is parallel to AiAi+3 (where
An+3=A3) and Ai+1Ai+2 < AiAi+3, we see Bi Finally, to get such a quadratic, we
For individual subscription for the next five issues for the
09-10 academic year, send us five stamped self-addressed is between Ai and Ai+3 on the segment compute the discriminant ∆= a2k2−4bk.
envelopes. Send all correspondence to:
AiAi+3. In particular, Bi is inside By taking a = 2 and b = 1, we have
Dr. Kin-Yin LI, Math Dept., Hong Kong Univ. of Science A1A2⋯An. (2k−2)2 < ∆= 4k2−4k < (2k−1)2.
and Technology, Clear Water Bay, Kowloon, Hong Kong
Fax: (852) 2358 1643 Next the coordinates of Bi are This leads to roots r, s irrational and
Email: makyli@ust.hk (xi+2−xi+1+xi, yi+2−yi+1+yi), both of 1 2k − Δ
© Department of Mathematics, The Hong Kong University which are integers. <s= < 1.
of Science and Technology 2 2
Mathematical Excalibur, Vol. 15, No. 3, Oct. 10-Dec. 10 Page 2

In the next example, we will need Example 4. (2001 IMO Proposal by Great So u=x=a2+b2, w= x+z =a2+b2+ ac+bd
to compute the exponent e of a prime Britain) Let ABC be a triangle with = a(a+c) + b(b+d) and v = y−u+2w =
number p such that pe is the largest centroid G. Determine, with proof, the (a+c)2+(b+d)2. This completes the
power of p dividing n!. The formula is position of the point P in the plane of ABC proof of the first statement.
such that
⎡n⎤ ⎡ n ⎤ ⎡ n ⎤ For the second statement, we have
e = ⎢ ⎥ + ⎢ 2 ⎥ + ⎢ 3 ⎥ + L. AP·AG+BP·BG+CP·CG
⎣ p⎦ ⎣ p ⎦ ⎣ p ⎦
is minimum, and express this minimum z 2 = (2n)!(2n)(2n −1)L1
Basically, since n!=1×2×⋯×n, we first value in terms of the side lengths of ABC. = (2n)!( p − (2n +1))L( p − 4n)
factor out p from numbers between 1 to
Solution. (Due to the late Professor ≡ (−1) 2n (4n)!= ( p −1)!≡ −1 (mod p),
n that are divisible by p (this gives [n/p]
factors of p), then we factor out another Murray Klamkin) Use a vector system
with the origin taken to be the centroid of where the last congruence is by
p from numbers between 1 to n that are Wilson’s theorem. This implies z2+1 is
divisible by p2 (this give [n/p2] more ABC. Denoting the vector from the origin
to the point X by X, we have a multiple of p, i.e. z2+1=py for some
factors of p) and so on. positive integer y. By the first
Example 3. (1983 and 1991 IMO
AP·AG+BP·BG+CP·CG statement, we see p =a2+b2 for some
Proposal by USSR) Let an be the last = |A−P||A|+|B−P||B|+|C−P||C| positive integers a and b.
nonzero digit (from left to right) in the ≥ |(A−P)·A|+|(B−P)·B|+|(C−P)·C|
decimal representation of n!. Prove Example 6. (1997 IMO Proposal by
= |A|2|+|B|2+|C|2 (since A+B+C=0) Russia) An infinite arithmetic
that the sequence a1, a2, a3, … is not
periodic after a finite number of terms = (BC2+CA2+AB2)/3. progression whose terms are positive
(equivalently 0.a1a2a3… is irrational). integers contains the square of an
Equality holds if and only if integer and the cube of an integer.
Solution. Assume beginning with the Show that it contains the sixth power of
|A−P||A|=|(A−P)·A|,
term aM, the sequence becomes an integer.
periodic with period t. Then for m ≥ M, |B−P||B|=|(B−P)·B|
we have am+t = am. To get a Solution. Let a be the first term and d
and |C−P||C|=|(C−P)·C|, be the common difference. We will
contradiction, we will do it in steps.
which is equivalent to P is on the lines GA, prove by induction on d. If d=1, then
Step 1. For every positive integer k, the terms are consecutive integers,
GB and GC, i.e. P=G.
(10k)! = (10k−1)!×10k implies hence the result is true. Next, suppose
a10k = a10k −1. The next example is a proof of a theorem d>1. Let r = gcd(a,d) and h=d/r, then
Step 2. We can get integers k > m ≥ M of Fermat. It is (the contrapositive of) an gcd(a/r,h)=1. We have two cases.
such that 10k−10m is a multiple of t as infinite descent argument that Fermat
follow. We factor t into the form 2r5sw, might have used. Case 1: gcd(r,h) = 1. Then gcd(a,h)=1.
where w is an integer relatively prime Since there exist x2 and y3 in the
Example 5. (1978 IMO Proposal by progression, so x2 and y3 ≡ a (mod d).
to 10. By Euler’s theorem, 10φ(w)−1 is a
France) Prove that for any positive Since h divides d, x2 and y3 ≡ a (mod h).
number divisible by w. Choose m =
integers x, y, z with xy−z2=1 one can find From gcd(a,h)=1, we get gcd(y,h)=1.
max{M,r,s} and k = m + φ(w). Then
nonnegative integers a, b, c, d such that Then there exists an integer t such that
10k−10m =2m5m(10φ(w)−1) is a multiple
x=a2+b2, y=c2+d2 and z=ac+bd. Set z = ty ≡ x (mod h). So
of t, say 10k−10m =ct for some integer c.
(2n)! to deduce that for any prime number
p=4n+1, p can be represented as the sum t6a2 ≡ t6y6 ≡ x6 ≡ a3 (mod h).
Step 3. Let n = 10k − 1 + ct. By
periodicity, we have of squares of two integers. Since gcd(a,h)=1, we may cancel a2 to
an = a10 k −1 = a10 k = an +1. Solution. We will prove the first statement get t6 ≡ a (mod h).
Let an=d, that is the last nonzero digit by induction on z. If z=1, then (x,y) = (1,2) Since gcd(r,h)=1, there exists an
of n! is d. Since (n+1)!=(n+1)×n! and or (2,1) and we take (a,b,c,d) = (0,1,1,1) or integer k such that kh ≡ −t (mod r).
the last nonzero digit of n+1= (1,1,0,1) respectively. Then we have (t+kh)6 ≡ 0 ≡ a (mod r)
2×10k−10m is 9, we see an+1=an implies Next for integer w > 1, suppose cases z = 1 and also (t+kh)6 ≡ a (mod h). Since
the units digit of 9d is d. Checking d=1 to w−1 are true. Let positive integers u,v, gcd(r,h)=1 and rh=d, we get (t+kh)6 ≡ a
to 9, we see only d = 5 is possible. So n! w satisfy uv−w2=1 with w>1. Note u=v (mod d). Hence, (t+kh)6 is in the
ends in 50…0. leads to w=0, which is absurd. Also u=w progression.
Step 4. By step 3, we see the prime leads to w=1, again absurd. Due to
Case 2: gcd(r,h) > 1. Let p be a prime
factorization of n! is of the form 2r5sw symmetry in u, v, we may assume u < v.
dividing gcd(r,h). Then p divides r,
Let x=u, y=u+v−2w and z=w−u. Since
with w relatively prime to 10 and s ≥ which divides a and d. Let pm be the
r+1> r. However, uv = w2+1 > w2 = uv−1 > u2−1, greatest power of p dividing a and pn be
so y ≥ 2(uv)1/2−2w > 0 and z = w−u > 0. the greatest power of p dividing d.
⎡n⎤ ⎡ n ⎤ ⎡ n ⎤ Since d = rh, p divides h and gcd(a,d) =
r = ⎢ ⎥ + ⎢ 2 ⎥ + ⎢ 3 ⎥ +L Next we can check xy−z2 = uv−w2 = 1. By
⎣2⎦ ⎣2 ⎦ ⎣2 ⎦ r, we see n > m ≥ 1.
inductive hypothesis, we have
⎡n⎤ ⎡ n ⎤ ⎡ n ⎤
> ⎢ ⎥ + ⎢ 2 ⎥ + ⎢ 3 ⎥ + L = s.
⎣ 5 ⎦ ⎣5 ⎦ ⎣5 ⎦ x = a2+b2, y = c2+d 2, z = ac+bd. (continued on page 4)
This is a contradiction and we are done.
Mathematical Excalibur, Vol. 15, No. 3, Oct. 10-Dec. 10 Page 3

Problem Corner ***************** where the inequality can be checked by


Solutions cross-multiplication. For x > 0, define
We welcome readers to submit their **************** x3/ 2 5
solutions to the problems posed below f ( x) = − ln x.
Problem 351. Let S be a unit sphere with 1 + x1 / 2 8
for publication consideration. The
solutions should be preceded by the center O. Can there be three arcs on S Its derivative is
solver’s name, home (or email) address such that each is a 300° arc on some circle
( x − 1)(8 x 3 / 2 + 20 x + 15 x1 / 2 + 5)
and school affiliation. Please send with O as center and no two of the arcs f ' ( x) = .
8 x( x + 1) 2
submissions to Dr. Kin Y. Li, intersect?
Department of Mathematics, The Hong This shows f(1)=1/2 is the minimum
Solution. Andy LOO (St. Paul’s Co-ed
Kong University of Science & College). value of f, since f’(x) < 0 for 0 < x <1
Technology, Clear Water Bay, Kowloon, and f’(x)>0 for x > 1. Then by (*),
Hong Kong. The deadline for sending The answer is no. Assume there exist
a 2bc a3 / 2 3 5 3
solutions is January 14, 2011. three such arcs l1, l2 and l3. For k=1,2,3, ∑ ≥∑
bc + 1 cyclic1 + a
1/ 2
≥ + ln abc ≥ .
2 8 2
let Ck be the unit circle with center O that cyclic

Problem 356. A and B alternately lk is on. Since lk is a 300° arc on Ck, every
Other commended solvers: Samuel
color points on an initially colorless point P on Ck is on lk or its reflection point Lilό ABDALLA (ITA-UNESP, São
plane as follow. A plays first. When A with respect to O is on lk. Let Pij and Pji be Paulo, Brazil), CHAN Chiu Yuen
takes his turn, he will choose a point the intersection points of Ci and Cj. (Since Oscar (Wah Yan College Hong Kong),
not yet colored and paint it red. When Pij and Pji are reflection points with Ozgur KIRCAK (Jahja Kemal
B takes his turn, he will choose 2010 respect to O, if Pij does not lie on both li College, Skopje, Macedonia), LAM
Lai Him (HKUST Math UG Year 2),
points not yet colored and paint them and lj, then Pji will be on li and lj, Andy LOO (St. Paul’s Co-ed College),
blue. When the plane contains three contradiction.) So we may let Pij be the LI Pak Hin (PLK Vicwood K. T.
red points that are the vertices of an point on li and not on lj and Pji be the point Chong Sixth Form College), Salem
equilateral triangle, then A wins. on lj and not on li. MALIKIĆ (Student, University of
Following the rules of the game, can B Sarajevo, Bosnia and Herzegovina),
Now P21 and P31 are on C1 and outside of NG Chau Lok (HKUST Math UG Year
stop A from winning? 1), Thien NGUYEN (Luong The Vinh
l1, so ∠P21OP31< 60°. Hence the length of
High School, Dong Nai, Vietnam), O
Problem 357. Prove that for every arcs P21P31 and P12P13 are equal and are Kin Chit Alex (GT(Ellen Yeung)
positive integer n, there do not exist less than π/3 (and similarly for ∠P32OP12, College), Carlo PAGANO (Università
four integers a, b, c, d such that ad=bc ∠P13OP23 and their arcs). Denote the di Roma “Tor Vergata”, Roma, Italy),
and n2 < a < b < c < d < (n+1)2. distance (i.e. the length of shortest path) Paolo PERFETTI (Math Dept,
between P and Q on S by d(P,Q). We have Università degli studi di Tor Vergata
Roma, via della ricerca scientifica,
Problem 358. ABCD is a cyclic π = d ( P12 , P21) Roma, Italy), Karatapanis SAVVAS
quadrilateral with AC intersects BD at (3rd Senior High School of Rhoades,
P. Let E, F, G, H be the feet of ≤ d ( P12 , P32 ) + d ( P32 , P31) + d ( P31, P21) Greece), TRAN Trong Hoang Tuan
perpendiculars from P to sides AB, BC, < π /3+π /3+π /3 = π, John (Bac Lieu Specialized Secondary
CD, DA respectively. Prove that lines School, Vietnam), WONG Chi Man
which is absurd. (CUHK Info Engg Grad), WONG Sze
EH, BD, FG are concurrent or are Nga (Diocesan Girls’ School), WONG
parallel. Other commended solvers: LI Pak Hin Tat Yuen Simon and POON Lok Wing
(PLK Vicwood K. T. Chong Sixth Form (Carmel Divine Grace Foundation
Problem 359. (Due to Michel College). Secondary School) and Simon YAU.
BATAILLE) Determine (with proof) all
Problem 353. Determine all pairs (x, y)
real numbers x,y,z such that x+y+z ≥ 3 Problem 352. (Proposed by Pedro
of integers such that x5−y2=4.
and Henrique O. PANTOJA, University of
Lisbon, Portugal) Let a, b, c be real Solution. Ozgur KIRCAK (Jahja
x3 + y3 + z3 + x4 + y4 + z4 ≤ 2(x2 + y2 + z2 ). numbers that are at least 1. Prove that Kemal College, Skopje, Macedonia),
LI Pak Hin (PLK Vicwood K. T.
Problem 360. (Due to Terence ZHU, a 2bc b 2ca c 2 ab 3 Chong Sixth Form College), Carlo
+ + ≥ .
Affiliated High School of Southern bc + 1 ca + 1 ab + 1 2 PAGANO (Università di Roma “Tor
China Normal University) Let n be a Vergata”, Roma, Italy), Anderson
Solution. D. Kipp JOHNSON (Valley TORRES (São Paulo, Brazil) and
positive integer. We call a set S of at Catholic School, Teacher, Beaverton,
least n distinct positive integers a Ghaleo TSOI Kwok-Wing (Univer-
Oregon, USA). sity of Warwick, Year 1).
n-divisible set if among every n
elements of S, there always exist two of From a 2 bc ≥ bc ≥ 1, we get Let x, y take on values −5 to 5. We see
them, one is divisible by the other. x5≡ 0, 1 or 10 (mod 11), but y2 +4 ≡2, 4,
a 2 bc a 2 bc a 2 bc 3 5, 7, 8 or 9 (mod 11). Therefore, there
Determine the least integer m (in terms ∑
cyclic
≥∑ =∑
bc + 1 cyclic 2 bc cyclic 2
≥ .
2 can be no solution.
of n) such that every n-divisible set S
with m elements contains n integers, Moreover, we will prove the stronger fact: Other commended solvers: Andy LOO
one of them is divisible by all the if a, b, c > 0 and abc≥1, then the inequality (St. Paul’s Co-ed College).
remaining n−1 integers. still holds. From k = abc ≥ 1, we get
Problem 354. For 20 boxers, find the
a 2bc ka 3 / 2 a3 / 2 , (*)
= ≥ least number n such that there exists a
k + a1 / 2 1 + a
1/ 2
bc + 1
Mathematical Excalibur, Vol. 15, No. 3, Oct. 10-Dec. 10 Page 4

schedule of n matches between pairs of D1 Problem 5. Let f(x) and g(x) be strictly
A
them so that for every three boxers, two D increasing linear functions from ℝ to ℝ
of them will face each other in one of such that f(x) is an integer if and only if
the matches. g(x) is an integer. Prove that for any
C C1
B real number x, f(x) − g(x) is an integer.
Solution. LI Pak Hin (PLK Vicwood
K. T. Chong Sixth Form College) and E
Andy LOO (St. Paul’s Co-ed College). B1 Problem 6. In acute ⊿ABC, AB > AC.
Let M be the midpoint of side BC. The
Among the boxers, let A be a boxer that Now if lines BB1, CC1 and DD1 concur at
exterior angle bisector of ∠BAC meets
will be in the least number of matches, E, then (2) can be restated as ∠AB1E
ray BC at P. Points K and F lie on line
say m matches. For the 19−m boxers =∠AC1E =∠AD1E. These imply A, B1, C1,
PA such that MF⊥BC and MK⊥PA.
that do not have a match with A, each D1, E are concyclic. So AB1C1D1 is cyclic.
Prove that BC2 = 4PF·AK.
pair of them with A form a triple. Since Then by similarity, ABCD is cyclic.
A doesn’t play them, every one of these F
D1 K A
(19−m)(18−m)/2 pairs must play each A D
other in a match by the required
condition.
O O1 C C1
For the m boxers that have a match B B M C P
with A, each of them (by the minimal F
condition on A) has at least m matches. B1
Problem 7. Let n be an integer greater
Since each of these matches may be than or equal to 3. For a permutation p
counted at most twice, we get at least For the converse, suppose ABCD is cyclic,
= (x1, x2, …, xn) of (1,2,…,n), we say xj
(m+1)m/2 more matches. So then AB1C1D1 is cyclic by similarity. Let
lies between xi and xk if i < j < k. (For
the two circumcircles intersect at A and F.
(19 − m)(18 − m) (m + 1)m example, in the permutation (1,3,2,4),
n≥ + Let O be the circumcenter of ABCD and
3 lies between 1 and 4, and 4 does not
2 2 O1 be the circumcenter of AB1C1D1. It
lie between 1 and 2.) Set S={p1,
= (m − 9) 2 + 90 ≥ 90. follows ⊿AOD and ⊿AO1D1 are similar.
p2,…,pm} consists of (distinct)
Hence ∠AOD = ∠AO1D1. From this we
Finally, n = 90 is possible by dividing permutations pi of (1,2,…,n). Suppose
get
the 20 boxers into two groups of 10 that among every three distinct
boxers and in each group, every pair is 1 1 numbers in {1,2,…,n}, one of these
∠AFD = ∠AOD = ∠AOD1 = ∠AFD1.
scheduled a match. This gives a total 2 2 numbers does not lie between the other
of 90 matches. two numbers in every permutation
This implies line DD1 passes through F.
pi∈S. Determine the maximum value
Similarly, lines BB1 and CC1 pass through
Other commended solvers: WONG of m.
F. Therefore, lines BB1, CC1 and DD1
Sze Nga (Diocesan Girls’ School).
concur. Problem 8. Determine the least odd
Problem 355. In a plane, there are two number a > 5 satisfying the following
Other commended solvers: LI Pak Hin
similar convex quadrilaterals ABCD conditions: There are positive integers
(PLK Vicwood K. T. Chong Sixth Form
and AB1C1D1 such that C, D are inside m1, m2, n1, n2 such that a = m12 + n12 ,
College).
AB1C1D1 and B is outside AB1C1D1
a 2 = m22 + n22 and m1 − n1 = m2 − n2 .
Prove that if lines BB1, CC1 and DD1
concur, then ABCD is cyclic. Is the
converse also true?
Olympiad Corner
(continued from page 1)
Solution. CHAN Chiu Yuen Oscar IMO Shortlisted Problems
(Wah Yan College Hong Kong) and Problem 3. Prove that for every given (continued from page 2)
LEE Shing Chi (SKH Lam Woo positive integer n, there exists a prime p
Memorial Secondary School). and an integer m such that Then pm divides a and d, hence all
(a) p≡5 (mod 6); terms a, a+d, a+2d, ⋯ of the
Since ABCD and AB1C1D1 are similar,
(b) p ∤n; progression. In particular, pm divides
we have
(c) n≡m3 (mod p). x2 and y3. Hence, m is a multiple of 6.
AB AC AD (1)
= = . Consider the arithmetic progression
AB1 AC1 AD1 Problem 4. Let x1, x2, …, xn (with n ≥ 2)
obtained by dividing all terms of a,
be real numbers such that
Also, ⊿ABC and ⊿AB1C1 are similar. a+d, a+2d,⋯ by p6. All terms are
Then ∠BAC =∠B1AC1. Subtracting x12 + x22 + L + xn2 = 1.
positive integers, the common
∠B1AC from both sides, we get ∠BAB1 Prove that difference is d/p6 < d and also contains
=∠CAC1. Similarly, ∠CAC1 =∠DAD1. 2
(x/p3)2 and (y/p2)3. By induction
Along with (1), these give us ⊿BAB1, n⎛ k ⎞ x2 ⎛ n − 1 ⎞2 n x 2 hypothesis, this progression contains a
⎜1 − ⎟ k ≤⎜
⊿CAC1 and ⊿DAD1 are similar. So ∑ ⎜ 2 ⎟
⎟ ∑ k. sixth power j6. Then (pj)6 is a sixth
⎝ ∑i =1 i ⎠
k =1
n
ix k ⎝ n + 1 ⎠ k =1 k
power in a, a+d, a+2d,⋯ and we are
∠AB1B =∠AC1C =∠AD1D. (2) done.
Determine when equality holds.

You might also like